How much interest will be earned on a $1600 investment at 7.72% compounded
quarterly for 9 years?

Answers

Answer 1

Answer:

A = $3,198.17

A = P + I where

P (principal) = $1,600.00

I (interest) = $1,598.17

Step-by-step explanation:

First, convert R as a percent to r as a decimal

r = R/100

r = 7.72/100

r = 0.0772 rate per year,

Then solve the equation for A

A = P(1 + r/n)nt

A = 1,600.00(1 + 0.0772/12)(12)(9)

A = 1,600.00(1 + 0.006433333)(108)

A = $3,198.17

Summary:

The total amount accrued, principal plus interest, with compound interest on a principal of $1,600.00 at a rate of 7.72% per year compounded 12 times per year over 9 years is $3,198.17.


Related Questions

Let Q(x, y) be the predicate "If x < y then x2 < y2," with domain for both x and y being R, the set of all real numbers.
a) When x = −2 and y = 1, is Q(x, y).
a. true
b. false
The hypothesis of Q(−2, 1) is__, which is___. The conclusion is___, which is____. Thus Q(−2, 1) is a conditional statement with a____hypothesis and a_____conclusion. So Q(−2, 1) is____.
b) Give values different from those in part (a) for which Q(x, y) has the same truth value as in part.
c) Give values different from those in part (c) for which Q(x, y) has the same truth values as in part.

Answers

Answer:

a) Q(-2,1) is false

b) Q(-5,2) is false

c)Q(3,8) is true

d)Q(9,10) is true

Step-by-step explanation:

Given data is [tex]Q(x,y)[/tex] is predicate that [tex]x<y[/tex] then [tex]x^{2} <y^{2}[/tex]. where [tex]x,y[/tex] are rational numbers.

a)

when [tex]x=-2, y=1[/tex]

Here [tex]-2<1[/tex] that is [tex]x<y[/tex]  satisfied. Then

[tex](-2)^{2}<1^{2}[/tex]

[tex]4<1[/tex] this is wrong. since [tex]4>1[/tex]

That is [tex]x^{2}[/tex][tex]>y^{2}[/tex] Thus [tex]Q(x,y)[/tex] [tex]=Q(-2,1)[/tex]is false.

b)

Assume [tex]Q(x,y)=Q(-5,2)[/tex].

That is [tex]x=-5, y=2[/tex]

Here [tex]-5<2[/tex] that is [tex]x<y[/tex] this condition is satisfied.

Then

[tex](-5)^{2}<2^{2}[/tex]

[tex]25<4[/tex] this is not true. since [tex]25>4[/tex].

This is similar to the truth value of part (a).

Since in both [tex]x<y[/tex] satisfied and [tex]x^{2} >y^{2}[/tex] for both the points.

c)

if [tex]Q(x,y)=Q(3,8)[/tex] that is [tex]x=3[/tex] and [tex]y=8[/tex]

Here [tex]3<8[/tex] this satisfies the condition [tex]x<y[/tex].

Then [tex]3^{2} <8^{2}[/tex]

[tex]9<64[/tex] This also satisfies the condition [tex]x^{2} <y^{2}[/tex].

Hence [tex]Q(3,8)[/tex] exists and it is true.

d)

Assume [tex]Q(x,y)=Q(9,10)[/tex]

Here [tex]9<10[/tex] satisfies the condition [tex]x<y[/tex]

Then [tex]9^{2}<10^{2}[/tex]

[tex]81<100[/tex] satisfies the condition [tex]x^{2} <y^{2}[/tex].

Thus, [tex]Q(9,10)[/tex] point exists and it is true. This satisfies the same values as in part (c)

Please help someone! Very important!

Answers

Answer:

OH NO

Step-by-step explanation:

Step-by-step explanation:

oh srr, I'm Vietnamese and I don't know your

country's solution

but i think my answer is correct :((

Translate the sentence into an equation. The quotient of a number and 8 is 16. What is the solution to the equation? One-half 2 128 164

Answers

Answer:

128

Step-by-step explanation:

I did the math

Answer:

it's 2 :)

Step-by-step explanation:

quotient is a term used in division so you divide

16 divided by 8 is 2 <3

don't forget to say thanks if you got it right ! :)

One side of a REGULAR OCTAGON is 19 ft.
What is the PERIMETER of this octagon?
feet.
I

Answers

Perimeter: 19ft x 8 sides = 152ft

Explanation: Regular octagon means equivalent sides and more.

Answer:

152 ft.

Step-by-step explanation:

19*8 = 152

find the measure of one interior angle in a regular 23-gon

Answers

Answer: A polygon with 23 sides has a total of 3780 degrees. total interior angles = (n - 2)180°, where n is the number of sides.

The measure of one interior angle of the given regular polygon with 23 sides is 164.3 degrees

How to calculate the sum of the interior angle of a regular polygon?

The formula which is used to calculate the sum of the angle of regular polygon is given by

sum of the interior angles = ( n - 2 ) × 180 degrees

Where,

n is the number of sides

According to the given question.

We have a regular polygon with 23 sides.

⇒ n = 23

Therefore,

The sum of the interior angles of the given regular polygon

= (23-2) × 180

= 3,780 degrees

So, the measure of one interior angle = [tex]\frac{3780}{23} = 164.3 degrees[/tex]

Hence, the measure of one interior angle of the given regular polygon with 23 sides is 164.3 degrees.

Find out more information about interior angle of a regular polygon here:

https://brainly.com/question/22408868

#SPJ3

Mary has three baking pans. Each pan is 8" × 8" × 3". Which expression will give her the total volume of the pans?

Answers

Answer: An expression [tex]3 \times (8 \times 8 \times 3)[/tex] will give her the total volume of the pans.

Step-by-step explanation:

Given: Length = 8 inch

Width = 8 inch

Height = 8 inch

Formula to calculate the volume of rectangular pans is as follows.

[tex]Volume = length \times width \times height\\[/tex]

Substitute the values into above formula as follows.

[tex]Volume = length \times width \times height\\= 8 \times 8 \times 3 in^{3}\\= 192 in^{3}[/tex]

Therefore, volume of each pan is 192 cubic inch. As there are three baking pans so total volume of the pans is as follows.

[tex]3 \times 192 in^{3}\\= 576 in^{3}[/tex]

Thus, we can conclude that an expression [tex]3 \times (8 \times 8 \times 3)[/tex] will give her the total volume of the pans.

Solve using the zero-factor proper (13x + 7)(6x - 7) = 0​

Answers

Answer:

x = -7/13 and x = 7/6

Step-by-step explanation:

Using the zero factor property, set each factor equal to zero, and solve for x:

13x + 7 = 0

13x = -7

x = -7/13

Solve the other factor:

6x - 7 = 0

6x = 7

x = 7/6

So, the solutions are x = -7/13 and x = 7/6

Answer is x=7/13 and x7/6

someone asap

A small factory has 3 machines for producing protractors. The high speed machines produces 61% the protractors but 6% of its output is defective. The medium speed machine produces 24% of the protractors, of which 4%o are defective. The low speed machine, which has a defective rate of 2% produces the remainder.
{a) Draw a tree diagram.

(b) What is the probability of a protractor not being defective given it came from a low speed machine?

(c). Knowing that a protractor is defective, what ls the probability it came from the high speed machine?

Answers

It’s A because I did the test lmafo

prove by factorization √16×81=√16×√81

Answers

Answer:

√16×81 = 4×9 or √1296 = 36

√16×√81 = 4×9 = 36

Step-by-step explanation:

√81×16

We simplify 81 and 16 by prime factorisation (expressing a number as a product of its prime factors).

√81 = √3×3×3×3 = √9×9 = √9² = 9

√16 = √2×2×2×2 = √4×4 = √4² = 4

Write the missing power of 10 in each equation.

0.80 × [ ] = 8
[ ] × 0.002 = 20
0.04 × [ ]= 4
[ ]× 0.5 = 500

Answers

1. 10 or 10 to the first power
2. 10000 or 10 to the fourth power
3. 100 or 10 to the second power
4. 1000 or 10 to the third power

Suppose a 90% confidence interval for the mean salary of college graduates in a town in Mississippi is given by [$38,737, $50,463]. The population standard deviation used for the analysis is known to be $14,300.
a. What is the point estimate of the mean salary for all college graduates in this town?
Point estimate
b. Determine the sample size used for the analysis.
Sample size

Answers

Answer:

a. The point estimate was of $44,600.

b. The sample size was of 16.

Step-by-step explanation:

Confidence interval concepts:

A confidence interval has two bounds, a lower bound and an upper bound.

A confidence interval is symmetric, which means that the point estimate used is the mid point between these two bounds, that is, the mean of the two bounds.

The margin of error is the difference between the two bounds, divided by 2.

a. What is the point estimate of the mean salary for all college graduates in this town?

Mean of the bounds, so:

(38737+50463)/2 = 44600.

The point estimate was of $44,600.

b. Determine the sample size used for the analysis.

First we need to find the margin of error, so:

[tex]M = \frac{50463-38737}{2} = 5863[/tex]

Relating the margin of error with the sample size:

We have that to find our [tex]\alpha[/tex] level, that is the subtraction of 1 by the confidence interval divided by 2. So:

[tex]\alpha = \frac{1 - 0.9}{2} = 0.05[/tex]

Now, we have to find z in the Z-table as such z has a p-value of [tex]1 - \alpha[/tex].

That is z with a pvalue of [tex]1 - 0.05 = 0.95[/tex], so Z = 1.64.

Now, find the margin of error M as such

[tex]M = z\frac{\sigma}{\sqrt{n}}[/tex]

In which [tex]\sigma[/tex] is the standard deviation of the population and n is the size of the sample.

For this problem, we have that [tex]\sigma = 14300, M = 5863[/tex]. So

[tex]M = z\frac{\sigma}{\sqrt{n}}[/tex]

[tex]5863 = 1.645\frac{14300}{\sqrt{n}}[/tex]

[tex]5863\sqrt{n} = 1.645*14300[/tex]

[tex]\sqrt{n} = \frac{1.645*14300}{5863}[/tex]

[tex](\sqrt{n})^2 = (\frac{1.645*14300}{5863})^2[/tex]

[tex]n = 16[/tex]

The sample size was of 16.

Find the volume of the composite solid. Round your answer to the nearest
tenth.

Answers

Answer:

452.4 in³

Step-by-step explanation:

Volume half sphere = 1/2 volume sphere

Volume sphere = 4/3πr²

Volume = 4/3*3.14*27             Radius = 1/2(6) .     3³ = 27

Volume ≈ 113.1

Volume Cylinder = Area circle x hieght

Area circle = πr²

Area circle = 3.14 * 9

Area circle = 28.26

Volume cylinder = 28.26 * 12 = 339.12

339.12 + 113.1 = 452.22

Because of rounding it would be ≈ 452.4

If my answer is incorrect, pls correct me!

If you like my answer and explanation, mark me as brainliest!

-Chetan K

What is the area of the triangle in centimeters squared?​

Answers

112 centimeters squared

Explanation:
The formula for area of a triangle is:
1/2 • base • height

REMEMBER, use the height that is perpendicular to the base. If not, your answer will be wrong

Use formula with given dimension of this triangle:
1/2 • 14 • 16 = 112

FINAL ANSWER WITH UNIT OF MEASURMENT:
112 centimeters squared

Have a nice day, hope this helps!

3. A bicycle has wheels with a diameter of 622 mm. The

bicycle rolls forward and the wheel turns 5 radians.

How many millimeters forward did the bicycle move?

Distance = ( ) (622) = (15.708) ()

= (2.618)

וחתן

Answers

Answer:

1553.6mm

Step-by-step explanation:

Given data

Diameter = 622mm

Radius= 622/2= 311mm

Circumference= 2πr

Circumference= 2*3.142*311

Circumference= 1954.32

Each revolution the wheels will turn 1954.32mm

Now let us convert radian to turns

1 radian= 0.159155 turns

5 radians=  x turns

cross multiply

x= 5*0.159155

x=0.795 turns

If 1 turn will give 1954.32mm

0.795 turn will give  x

cross multiply

x= 0.795*1954.32

x=1553.6mm

in a class of 30 students, 5 have a brother and 23 have a sister. there are 2 students who have a brother and a sister. what is the probability that a student has a sister also has a brother

Answers

2/5

lmk if this helped :)

Answer:

2/23

Hope this helps!

D
76
21. In the last three basketball games,
Paula has scored 21, 25, and
15 points p. Which inequality does
not describe the numbers of points
she has scored?
Ap> 21
Bps 25
cp< 30
Dp 2 10

Answers

Answer:

A

Step-by-step explanation:

Note that

> means greater than

< means less than

≥ means greater than or equal to  

≤ less than or equal to  

for example : 2 < 3 means 2 is less than 3

3 >2 means 3 is greater than 2

Paula's 3 scores are greater than 15 and less than 26

p> 21 implies her score is greater than 21. this is wrong because there is a score of 15

what are the domain and range of this function?

Answers

Answer:

domain: all real numbers

range: {y | y ≥ 0}

there are 750 spectator in the stadium of which 420 are women and the rest are men​

Answers

Complete Question:

There are 750 spectator in the stadium of which 420 are women and the rest are men. What percent of the spectators are women?

Answer:

Percentage = 56%

Step-by-step explanation:

Given the following data;

Total number of people = 750

Number of women = 420

To find the percentage of women;

First of all, we would determine the number of male spectators (men);

Number of men = Total number of people - Number of women

Number of men = 750 - 420

Number of men = 330

Next, we find the percentage of women;

[tex] Percentage = \frac {420}{750} * 100 [/tex]

[tex] Percentage = \frac {42}{75} * 100 [/tex]

[tex] Percentage = 0.56 * 100 [/tex]

Percentage = 56%

Therefore, the percentage of the spectators that are women is 56%.

Which is the solution to the equation below? 4n+5=25-3n

Answers

Answer:

[tex]n = \frac{20}{7}[/tex]

Step-by-step explanation:

[tex]4n + 5 = 25 - 3n\\4n + 3n = 25 - 5 \\7n = 20\\\\n = \frac{20}{7}[/tex]

Answer:

n = 20/7

Step-by-step explanation:

4n+5=25-3n

Add 3n to each side

4n+3n+5=25-3n+3n

7n +5 = 25

Subtract 5 from each side

7n+5-5=25-5

7n = 20

Divide by 7

7n/7 = 20/7

n = 20/7

anyone know the answer to this question ?

Answers

Question:

c=3x+80  

R=12x- 0.02x^2

R= revenue  

C=cost  

X=items sold  

A) 9x-(0.2x^2+80)

B) x(9-0.2x)-80  

C) x(9-0.02x)

D) 9x-0.02x^2+80

Answer:

c) x(9-0.2x)

is the correct answer

PLZ MARK BRAINLIEST

6x+2y=
6x+2y=
\,\,16
16
2x-2y=
2x−2y=
\,\,32
32

Answers

Answer:

x = 6, y = -8

Step-by-step explanation:

First we take the equations side by side and put them like this:

 6x + 2y = 16

+ 2x - 2y = 32

8x = 48

Then solve for x.

x = 6

Then insert x back into either equation

2(5) - 2y = 32

10 - 2y = 32

solve for y,

y = -10

Simplify the following expression.

29.718 - 29.63

Answers

Answer:

0.88 is the correct answer

Hi! Could someone check this questions for me and if I have anything wrong tell what I have wrong please? That would be highly appreciated!!

Answers

Answer:

Both are correct

I'm only checking the first page

Step-by-step explanation:

Answer:

Step-by-step explanation:

The 3rd page both are wrong the correct answers are 6 degrees C and Ice would form on the top of the lake like it does now.

The 4th page the bottom one is wrong the correct answer is -22.

which number represents 4%​

Answers

Answer:

4

Step-by-step explanation:

true or false m angle 5 is greater than m angle 8​

Answers

Answer:

True

Step-by-step explanation:

You can see that Angle 5 is slightly greater than 90°, so it is obtuse

Angle 8 is less than 90°, and is acute

Since obtuse angles are larger than acute angles, Angle 5 is greater than Angle 8.

If my answer is incorrect, pls correct me!

If you like my answer and explanation, mark me as brainliest!

-Chetan K

Which expression can be used to convert 22 Australian dollars to US dollars? Assume 1.2 Australian dollars equals 1 US dollar

Answers

Answer: 26.4

Step-by-step explanation: 1.2x22=

Find the value of x in each case:

Answers

Answer:

x = 19

Step-by-step explanation:

∠ YWX = ∠ ZYW = 2x ( alternate angles )

The sum of the 3 angles in Δ WXY = 180° , that is

3x - 5 + 2x + 90 = 180

5x + 85 = 180 ( subtract 85 from both sides )

5x = 95 ( divide both sides by 5 )

x = 19

The Coordinate Plane
BRE
-2
В
The midpoint of AB = ([?],[ ])

Answers

Answer:

0,0

Step-by-step explanation:

In the video, you saw that Michael used a budget to make sure he pays bills when they’re due. What are some other reasons someone would want to create a budget?

Answers

Someone might want to create a budget to treat themselves later with their saved money. Another reason it is good to have a budget is to not max out your credit card.

A quantity P is an exponential function of time t. Use the given information about the function P = P0e^{kt} to find values for the parameters k and P0.
​P=40 when t=4 and P=50 when t=3.

Answers

Answer:

P = 40 x 4

= 160

P = 50 x 3

= 150

Other Questions
Divide x3 + 5x2 + 5x 3 by x + 3 100 PointsSolve x^2 -6x=-5 by completing the square. Show all work for the steps below. (a) For x^2 -6x+c+-5+c, what value of c is used to complete the square? (b) Substitute the value for c in Part 2(a). Then complete the square to rewrite the equation as the square of a binomial. (c) Solve for x. 1. A quadratic equation can have all of the following number of solutions exceptd,b.ZeroOneInfiniteTwoc.a Can someone complete the story?i want ideas species Ispecies IIspecies IIIspecies IV Twelve is added to 5x and equals - 3. Find the value of x. Choose the best choice for the questionbelow.20. Which sentence is written correctly?A. The book or the pen are in the drawer.B. She and her friend is going to the mall.C. Mary and her brother wishes that they could go.D. My father or my mother is giving us a ride toschool. oThe growth of Ghana, Mali, and Songhai was based onSelect one:ofO a. their conversion to Islamb. their control of trade routes in Saharan Africac. their exportation of cotton, indigo, and sugarO d. their control of the Trans-Atlantic slave trade HELP) of the following WOULD YOU NOT FIND IN THE DESERT TORNADODRY HUMIDITYHURRICANENONE OF THE ABOVE Companies must disclose when they give products to online reviewers. rue or false 6 tons equals how many lbs? Safety Issues for Operators of Oil and Gas Exploiting Equipment when working off rigs Schools should not be able to punish students for speech made outside of school because? please write a reasonable paragraph explaning why. Helpppp plzzzzz asapppppppppppppppppp find the ratio in which the point (3,y) divides the line joining the point (9,8) and (-4,-6) hence find the value of y How come no one except me (in texas) cares about masks. there are new dangorus veriants that the vacene can not protect you from. Why do potato plants no longer need to use glucose from starch in respiration once they have grown above ground? The slope of this line is (-12,10) and (-10,6) Please help the question is in the picture. ***no links please:)****